La ricerca ha trovato 89 risultati

da Rho33
01 ago 2016, 18:38
Forum: Combinatoria
Argomento: Galileiana 2014 5
Risposte: 2
Visite : 2796

Re: Galileiana 2014 5

Fun fact: cercando in internet ho scoperto che il problema era già uscito su olifis (il $3$ è scambiato con un $9$, ma cambia poco), dove il propositore (che è anche utente di questo forum) ha ricevuto risposte, per così dire, sbagliate/ senza senso (sono certo del risultato ottenuto), mentre il suo...
da Rho33
01 ago 2016, 18:23
Forum: Glossario e teoria di base
Argomento: Birapporto all'infinito?
Risposte: 9
Visite : 11219

Re: Birapporto all'infinito?

Quindi, se non sto dicendo grosse scemenze, per definire in modo rigoroso, dovrei dimostrare che la funzione birapporto (?) ammette limite? :?: In caso, come?
da Rho33
01 ago 2016, 04:46
Forum: Glossario e teoria di base
Argomento: Birapporto all'infinito?
Risposte: 9
Visite : 11219

Re: Birapporto all'infinito?

Grazie mille ad entrambi per i due metodi diversi! :D
da Rho33
01 ago 2016, 04:45
Forum: Combinatoria
Argomento: Galileiana 2014 5
Risposte: 2
Visite : 2796

Galileiana 2014 5

In un gioco tradizionale armeno un giocatore gioca contro il banco lanciando due dadi. Se la somma dei risultati che appaiono sulle facce superiori vale $7$ o $11$ il giocatore vince, se invece la somma vale $2, 3 $ o $ 12$ vince il banco. Nel caso in cui al primo lancio il giocatore ottenga un risu...
da Rho33
31 lug 2016, 18:07
Forum: Glossario e teoria di base
Argomento: Birapporto all'infinito?
Risposte: 9
Visite : 11219

Birapporto all'infinito?

Mentre cercavo qualche quaterna di punti figa che avesse un birapporto carino (magari utile), mi è venuta in mente questa: $$(M,P_{\infty};A,B)$$ Dove $A,B$ sono due punti su una retta, $M$ è il loro punto medio e $P_{\infty}$ è un punto all'infinito sulla retta $AB$. Se io calcolo il birapporto (se...
da Rho33
30 lug 2016, 03:36
Forum: Glossario e teoria di base
Argomento: Fattorizzazione unica
Risposte: 1
Visite : 6190

Fattorizzazione unica

Come da titolo, mi chiedevo se esiste un modo (che non coinvolga troppa teoria alle spalle) per dimostrare che un dato insieme è UFD. Quando uno lavora in qualche anello strano vorrebbe sapere subito se esso sia UFD o meno (anziché provare a trovare un controesempio, che magari richiede moltissimo t...
da Rho33
30 lug 2016, 03:12
Forum: Teoria dei Numeri
Argomento: $\mu(n)=\mu(n+1)=\mu(n+2)=1$
Risposte: 12
Visite : 5340

Re: $\mu(n)=\mu(n+1)=\mu(n+2)=1$

Uhm, anche se ho già risposto nell'altro topic, mi ero dimenticato di ringraziare per il tuo articolo, sembra davvero ben fatto (anche se ovviamente dovrò leggere ogni pagina almeno 10 volte per capirla a fondo :oops: ). Un ulteriore dubbio: sapresti dirmi se ci sono densità asintotiche di insiemi p...
da Rho33
29 lug 2016, 17:44
Forum: Teoria dei Numeri
Argomento: $d^\star(X)=1$ e $d_\star(X)=0$
Risposte: 8
Visite : 3687

Re: $d^\star(X)=1$ e $d_\star(X)=0$

Mah, io avevo pensato a costruirlo in modo molto rozzo (e informale, purtroppo) :oops: Sostanzialmente si tratta di costruire un insieme con buchi arbitrariamente grandi e piccoli sui naturali. Fissiamo un primo $p$ (ma si fa con qualsiasi altro numero) e costruiamo in questo modo: 1) $1$ lo metto n...
da Rho33
27 lug 2016, 01:27
Forum: Teoria dei Numeri
Argomento: $\mu(n)=\mu(n+1)=\mu(n+2)=1$
Risposte: 12
Visite : 5340

Re: $\mu(n)=\mu(n+1)=\mu(n+2)=1$

Aaaallora, in realtà non conoscevo né la definizione di densità asintotica, nè quel risultato (o meglio, forse avevo letto qualcosa da qualche parte in qualche momento lontano, ma non ci avevo fatto caso e quindi avevo rimosso completamente :oops: ). Chiaramente con quel risultato il problema muore ...
da Rho33
26 lug 2016, 18:59
Forum: Teoria dei Numeri
Argomento: $\mu(n)=\mu(n+1)=\mu(n+2)=1$
Risposte: 12
Visite : 5340

Re: $\mu(n)=\mu(n+1)=\mu(n+2)=1$

Poco fa ho pensato a questo mini-corollario (è per caso anche questo un problema aperto?), ma sembra molto molto difficile da dimostrare(senza la congettura di Dickson ovviamente) :oops: Qualcuno può confermare ed in caso tarare il livello di difficoltà? (in modo da non spenderci i giorni a vuoto :o...
da Rho33
26 lug 2016, 18:52
Forum: Geometria
Argomento: PreIMO 2015 G7
Risposte: 3
Visite : 3121

Re: PreIMO 2015 G7

Innanzitutto, grazie mille per il link! :D Inoltre, è assurdo che abbiamo usato le stesse notazioni per il lemma :o :o (che ora so chiamarsi in due modi diversi, tra l'altro :lol: ). C'è però un problema, l'utente di Aops liquida in due parole il caso mancante (cioè $I_1,I_3,X$ allineati) dicendo ch...
da Rho33
26 lug 2016, 17:58
Forum: Geometria
Argomento: PreIMO 2015 G7
Risposte: 3
Visite : 3121

PreIMO 2015 G7

Questo problema mi ha tormentato per due settimane di fila, o forse anche di più. Cercavo disperatamente una soluzione diversa da quella del video (con tutti quei conti di angoli e trigonometria :evil: ), ma non riuscivo a trovarla. Un po' di progressi li ho fatti, ma ancora non riesco a concludere ...
da Rho33
26 lug 2016, 16:54
Forum: Teoria dei Numeri
Argomento: Quattro Quadrati
Risposte: 3
Visite : 2454

Re: Quattro Quadrati

Oops, queste sono tutte le scemenze che scrivo alle $3$ di notte :oops: :oops: Cerco di spiegare/ aggiustare/ riempire i buchi che ho lasciato. Innanzitutto $p=2n+1$. Consideriamo i due insiemi: $$R:= \{a^2 \mid a=0,1, \dots , \frac {p-1}{2} \} \ \ , \ \ S:= \{ -b^2-1 \mid b=0,1, \dots , \frac {p-1}...
da Rho33
26 lug 2016, 16:14
Forum: Teoria dei Numeri
Argomento: $\mu(n)=\mu(n+1)=\mu(n+2)=1$
Risposte: 12
Visite : 5340

Re: $\mu(n)=\mu(n+1)=\mu(n+2)=1$

Oops, sì, l'ho scritta a caso quella frase. Intendevo: $30$ è prodotto di un numero dispari di primi (per la precisione di $3$, ovvero $2,3,5$ ) quindi $\mu (30)=-1$, ma dato che $\mu (p)=-1$ (poiché vi è un numero dispari di primi, cioè $1$), allora $\mu (30 \cdot p)=1$
da Rho33
25 lug 2016, 18:53
Forum: Teoria dei Numeri
Argomento: $\mu(n)=\mu(n+1)=\mu(n+2)=1$
Risposte: 12
Visite : 5340

Re: $\mu(n)=\mu(n+1)=\mu(n+2)=1$

Non avevo letto l'articolo di Wiki e non avevo compreso la potenza di questa congettura :o :o La parte c) è ovviamente uccisa all'istante: Per Dickson abbiamo che vi sono infiniti $a$ tali che : $$870a+1, \ \ 899a+1, \ \ 930a+1$$ sono tutti primi. Ma allora considerando i tre interi consecutivi $\eq...